Solving Inequalities and graphing them.

Solving Inequalities And Graphing Them.

Answers

Answer 1

Answer:

I used to know this not anymore sry g


Related Questions

Manipulate the radius and height of the cone, setting different values for each. Record the radius, height, and exact volume of the cone (in terms of π). The first one has been done for you. Also calculate the decimal value of the volume, and verify that it matches the volume displayed by the tool. (You might see some discrepancies in the tool due to rounding of decimals.)

Answers

Answer:

The decimal value of the volume already given= 1885.2 unit³

For radius 11 unit height 12 unit

Volume= 484π unit³

Volume= 1520.73 unit ³

For radius 4 unit height 6 unit

Volume= 32π unit³

Volume= 100.544 unit³

For radius 20 unit height 15 unit

Volume= 2000π unit³

Volume= 6284 unit³

Step-by-step explanation:

The decimal value of the volume already given= 600π

The decimal value of the volume already given= 600*3.142

The decimal value of the volume already given= 1885.2 unit³

For radius 11 unit height 12 unit

Volume= πr²h/3

Volume= 11²*12/3 *π

Volume= 484π unit³

Volume= 1520.73 unit ³

For radius 4 unit height 6 unit

Volume = πr²h/3

Volume= 4²*6/3(π)

Volume= 32π unit³

Volume= 100.544 unit³

For radius 20 unit height 15 unit

Volume= πr²h/3

Volume= 20²*15/3(π)

Volume= 2000π unit³

Volume= 6284 unit³

Here's the right answer.

if 25% of a number is 75 find the number​

Answers

Answer:

x = 300

Step-by-step explanation:

of means multiply and is means equals

25% * x = 75

Change to decimal form

.25x = 75

Divide each side by .25

.25x/.25 = 75/.25

x = 300

Answer:

300

Step-by-step explanation:

Assume the unknown value is 'Y'

75 = 25% x Y

75 = 25/100 x Y

Multiplying both sides by 100 and dividing both sides of the equation by 25 we will arrive at:

Y = 3 x 100/25

Y = 300%

Answer: 75 is 25 percent of 300

Astrid is in charge of building a new fleet of ships. Each ship requires 404040 tons of wood, and accommodates 300300300 sailors. She receives a delivery of 444 tons of wood each day. The deliveries can continue for 100100100 days at most, afterwards the weather is too bad to allow them. Overall, she wants to build enough ships to accommodate at least 210021002100 sailors.

Answers

To build the fleet of ships, Astrid must consider each of the given rates (i.e.  the daily tons of wood, the sailors per ship, etc.). The available deliveries are enough to build ships that can accommodate at least 2100 sailors.

Given that:

Required quantities

[tex]Wood = 40\ tons[/tex]

[tex]Sailors = 300[/tex] per ship

Available quantities

[tex]Wood = 4\ tons[/tex] daily

[tex]Days = 100[/tex] at most

First, we calculate the total tons of woods Astrid can receive.

[tex]Total = Days \times Wood\ Available[/tex]

[tex]Total = 100 \times 4[/tex]

[tex]Total = 400\ tons[/tex] ---- in 100 days

Next, we calculate the number of ships that can be made from the 400 tons.

[tex]Ships = \frac{Total\ tons}{Wood\ Required}[/tex]

So, we have:

[tex]Ships = \frac{400}{40}[/tex]

[tex]Ships = 10[/tex]

This means that Astrid can build up to 10 ships

The number of sailors the ship can accommodate is:

[tex]Sailors = Ships \times Sailors\ per\ ship[/tex]

So, we have:

[tex]Sailors = 10 \times 300[/tex]

[tex]Sailors = 3000[/tex]

It means the 10 ships can accommodate 3000 sailors.

3000 sailors is greater than 2100 sailors.

So, we can conclude that she can build enough ship for the 2100 sailors.

Read more about

https://brainly.com/question/17174491

Answer:

280 tons

Step-by-step explanation:

:)


* Graph these numbers on a number line.
-5,3, -2,1
-5

Answers

-5,3,-2,1 on a number line

<-|----|----|----|----|----|----|----|----|->

-5 -2 0 1 3

ASAP PLEASE GIVE CORRECT ANSWER
In a rectangular coordinate system, what is the number of units in the distance from the origin to the point $(-15, 8)$? Enter your answer

Answers

distance of a point [tex](x,y)[/tex] from origin is $\sqrt{x^2+y^2}$

so distance is $\sqrt{(-15)^2+(8)^2}=\sqrt{225+64}=\sqrt{289}=17$

Answer:

Distance=17 units

Step-by-step explanation:

Coordinates of the origin: (0, 0)

Coordinates of the point in question: (-15, 8)

Distance formula for any two points [tex](x_1,y_1), (x_2,y_2)[/tex] on the plane:

[tex]distance=\sqrt{(x_2-x_1)^2+(y_2-y_1)^2} \\distance=\sqrt{(-15-0)^2+(8-0)^2}\\distance=\sqrt{(15)^2+(8)^2}\\distance=\sqrt{225+64} \\distance=\sqrt{289} \\distance=17[/tex]

A company makes nylon and canvas backpacks. The profit on a nylon backpack is $3 and the profit on a canvas backpack is $10. How many backpacks must the company sell to make a profit of more than $250? Write a linear inequality that describes the situation.

Answers

Answer:

3x +10y  is greater than or equal to 250.

Step-by-step explanation:

The question asks us to write an inequality which shows that both nylon and canvas added should be greater than or equal to 250.

Since we don't know the number of nylon backpacks and canvas backpacks the company makes, we used the variables "x" and "y" to represent the number of backpacks they made from each style.

Answer:

3n + 10c > 250

Step-by-step explanation:

I confirmed it in grandpoint

Translate the following phrase into an algebraic expression using the variable m. Do not simplify,
the cost of renting a car for one day and driving m miles if the rate is $39 per day plus 45 cents per mile

Answers

Answer:

y  = 0.45X + 39  

Find the volume of a pyramid with a square base, where the side length of the base is 17 in 17 in and the height of the pyramid is 9 in 9 in. Round your answer to the nearest tenth of a cubic inch.

Answers

Answer:

The volume of the pyramid is 867 inch^3

Step-by-step explanation:

Here in this question, we are interested in calculating the volume of a square based pyramid.

Mathematically, we can use the formula below to calculate the volume V of a square based pyramid.

V = a^2h/3

where a represents the length of the side of the square and h is the height of the pyramid

From the question, the length of the side of the square is 17 in while the height is 9 in

Plugging these values, we have ;

V = (17^2 * 9)/3 = 17^2 * 3 = 867 cubic inch

john always wears a shirt, pants, socks, and shoes. he owns 12 pairs of socks, 3 pairs of shoes, 5 pairs of pants, and 5 shirts. how many different outfits can john make? PLEASE ANSWER

Answers

Answer:

900 outfits

Step-by-step explanation:

You just have to multiply them all together :)

What is the value of x in the equation 3x-4y=65, when y=4?
x=13 1/4
x=21 2/3
x =23
x = 27

Answers

Hello there! :)

Answer:

[tex]\huge\boxed{x = 27}[/tex]

Given the equation:

3x - 4y = 65 where y = 4

Substitute in 4 for "y":

3x - 4(4) = 65

Simplify:

3x - 16 = 65

Add 16 to both sides:

3x - 16 + 16 = 65 + 16

3x = 81

Divide both sides by 3:

3x/3 = 81/3

x = 27.

Answer: x = 27

Explanation:

3x - 4y = 65
3x - 4(4) = 65
3x - 16 = 65
3x = 65 + 16
3x = 81
x = 81/3
x = 27

(x+3)(x-5)=(x+3)(x−5)=

Answers

Answer:

All real numbers are solutions. 0=0

Step-by-step explanation:

(x+3)(x−5)=(x+3)(x−5)

Step 1: Simplify both sides of the equation.

x2−2x−15=x2−2x−15

Step 2: Subtract x^2 from both sides.

x2−2x−15−x2=x2−2x−15−x2

−2x−15=−2x−15

Step 3: Add 2x to both sides.

−2x−15+2x=−2x−15+2x

−15=−15

Step 4: Add 15 to both sides.

−15+15=−15+15

0=0

All real numbers are solutions.

What does the law of cosines reduce to when dealing with a right angle

Answers

Answer:

It is reduced to the equation of the Theorem of Pythagoras.

Step-by-step explanation:

Any triangle can be modelled by this formula under the Law of Cosine:

[tex]b = \sqrt{a^{2}+c^{2}-2\cdot a\cdot c\cdot \cos B}[/tex]

Where:

[tex]a[/tex], [tex]b[/tex], [tex]c[/tex] - Side lengths, dimensionless.

[tex]B[/tex] - Angle opposed to the side [tex]b[/tex], measured in sexagesimal degrees.

Now, let suppose that angle B is a right angle (90º), so that b is a hypotenuse and a and c are legs. Hence:

[tex]\cos B = 0[/tex]

And the equation is reduced to the form of the Theorem of Pythagoras, that is to say:

[tex]b = \sqrt{a^{2}+c^{2}}[/tex]

Pick out the set of numbers that is not Pythagorean triple
9 40 46
16 30 34
10 24 26
50 120 130

Answers

Answer:

[tex]\huge\boxed{9,40,46}[/tex]

Step-by-step explanation:

Let's check it using Pythagorean Theorem:

[tex]c^2 = a^2 + b^2[/tex]

Where c is the longest sides, a and b are rest of the 2 sides

1) 9 , 40 , 46

=> [tex]c^2 = a^2 + b^2[/tex]

=> [tex]46^2 = 9^2 + 40^2[/tex]

=> 2116 = 81 + 1600

=> 2116 ≠ 1681

So, this is not a Pythagorean Triplet

2) 16, 30 and 34

=> [tex]c^2 = a^2 + b^2[/tex]

=> [tex]34^2 = 16^2 + 30^2[/tex]

=> 1156 = 256 + 900

=> 1156 = 1156

No need to check more as we've found the one which is not a Pythagorean Triplet.

Answer:

[tex] \boxed{ \huge{ \boxed{ \sf{ \blue{9 , \: 40 \:, 46 \: }}}}}[/tex]

Option A is the correct option.

Step-by-step explanation:

1. Let h , p and b are the hypotenuse , perpendicular and base of a right - angled triangle respectively.

From Pythagoras theorem,

[tex] \sf{ {h}^{2} = {p}^{2} + {b}^{2} }[/tex]

Here, we know that the hypotenuse is always greater than perpendicular and base,

h = 46 , p = 40 , b = 9

⇒[tex] \sf{ {46}^{2} = {40}^{2} + {9}^{2} }[/tex]

⇒[tex]2116 = 1600 + 81[/tex]

⇒[tex] \sf{2116  ≠ 1681}[/tex]

Thus , the relation [tex] \sf{ {h}^{2} = {p}^{2} + {b}^{2} }[/tex] is not satisfied by h = 46 , p = 40 , b = 9

So, The set of numbers 9 , 40 , 46 is not Pythagorean triple.

------------------------------------------------------

2. 16 , 30 , 34

h = 34 , p = 30 , b = 16

[tex] \sf{ {h}^{2} = {p}^{2} + {b}^{2} }[/tex]

⇒[tex] \sf{ {34}^{2} = {30}^{2} + {16}^{2} }[/tex]

⇒[tex] \sf{1156 = 900 + 256}[/tex]

⇒[tex] \sf{1156 = 1156}[/tex]

The relation [tex] \sf{ {h}^{2} = {p}^{2} + {b}^{2} }[/tex] is satisfied by the particular values of h , p and b i.e h = 34 , p = 30 , b = 16

So, the set of numbers 16 , 30 , 34 is a Pythagorean triple.

------------------------------------------------------

3. 10, 24 , 26

h = 26 , p = 24 , b = 10

[tex] \sf{ {h}^{2} = {p}^{2} + {b}^{2} }[/tex]

⇒[tex] \sf{ {26}^{2} = {24}^{2} + {10}^{2} }[/tex]

⇒[tex] \sf{676 = 576 + 100}[/tex]

⇒[tex] \sf{676 = 676}[/tex]

The relation [tex] \sf{ {h}^{2} = {p}^{2} + {b}^{2} }[/tex] is satisfied by the particular values of h , p and h i.e h = 26 , p = 24 , b = 10

So, the set of numbers 10, 24 , 26 is the Pythagorean triple.

-----------------------------------------------------

4. 50 , 120 , 130

h = 130 , p = 120 , b = 50

[tex] \sf{ {h}^{2} = {p}^{2} + {b}^{2} }[/tex]

⇒[tex] \sf{ {130}^{2} = {120}^{2} + {50}^{2} }[/tex]

⇒[tex] \sf{16900 = 14400 + 2500}[/tex]

⇒[tex] \sf{16900 = 16900}[/tex]

The relation [tex] \sf{ {h}^{2} = {p}^{2} + {b}^{2} }[/tex] is satisfied by the particular values of h , p and b i.e h = 130 , p = 120 , b = 50

So, the set of numbers 50, 120 , 130 is the Pythagorean triple.

-----------------------------------------------------

In this way, to satisfy the Pythagoras Theorem , the hypotenuse ( h ) , perpendicular ( p ) and the base ( b ) of a right - angles triangle should have the particular values in order. These values of h , p and b are called Pythagorean triple.

Hope I helped!

Best regards!!

Events A and B are mutually exclusive. Find the missing probability.


P(A) = 1/4 P(B) = 13/20 P(A or B) = ?


4/5


1/2


9/10


3/8

Answers

Answer:

Option C.

Step-by-step explanation:

It is given that,

[tex]P(A)=\dfrac{1}{4}[/tex]

[tex]P(B)=\dfrac{13}{20}[/tex]

It is given that events A and B are mutually exclusive. It means they have no common elements.

[tex]P(A\cap B)=0[/tex]

We know that,

[tex]P(A\ or\ B)=P(A\cup B)=P(A)+P(B)-P(A\cap B)[/tex]

On substituting the values, we get

[tex]P(A\cup B)=\dfrac{1}{4}+\dfrac{13}{20}-0[/tex]

[tex]P(A\cup B)=\dfrac{5+13}{20}[/tex]

[tex]P(A\cup B)=\dfrac{18}{20}[/tex]

[tex]P(A\cup B)=\dfrac{9}{10}[/tex]

Therefore, the correct option is C.

The P (A or B) should be [tex]\frac{9}{10}[/tex]

Given that,

P(A) = 1 by 4  P(B) = 13 by 20

Based on the above information, the calculation is as follows:

[tex]= \frac{1}{4} + \frac{13}{20}\\\\= \frac{5+13}{20} \\\\= \frac{18}{20}\\\\= \frac{9}{10}[/tex]

Learn more: https://brainly.com/question/17429689?referrer=searchResults

9(p−4)=−18 p= I am not great at math, please explain just a little bit

Answers

Answer:

[tex]\large \boxed{{p=2}}[/tex]

Step-by-step explanation:

9(p-4) = -18

Expand brackets.

9p -36 = -18

Add 36 on both sides.

9p -36 + 36 = -18 + 36

9p = 18

Divide both sides by 9.

(9p)/9 = 18/9

p = 2

Answer:

p = 2

Step-by-step explanation:

9(p - 4) = -18

You are solving for the variable, p. Note the equal sign, what you do to one side, you do to the other. Do the opposite of PEMDAS.

PEMDAS is the order of operation, and =

Parenthesis

Exponents (& Roots)

Multiplication

Division

Addition

Subtraction

~

First, divide 9 from both sides:

(9(p - 4))/9 = (-18)/9

(p - 4) = -18/9

p - 4 = -2

Isolate the variable, p. Add 4 to both sides:

p - 4 (+4) = -2 (+4)

p = -2 + 4

p = 4 - 2

p = 2

Check. Plug in 2 for p in the equation:

9(p - 4) = -18

9(2 - 4) = -18

9(-2) = - 18

-18 = -18.

~

Simplify the following expression.

Answers

Answer:

3x+11y-3

Step-by-step explanation:

Hey! So here is what you do to solve the problem-

Combine like terms:

(x) 5x-2x=3x

(y) 3y+8y=11y

(#) 7-10 =-3

So....

3x+11y-3 is your answer!

Hope this helps!:)

ABC is an equilateral triangle, solve y

Answers

Answer:

y is 60⁰

because all sides are equal

Answer:

60 degrees

Step-by-step explanation:

In an equilateral triangle, the angles are equiangluar and the sides are equal.

180 degrees in a triangle/3 sides =

= 60 degrees per side

Sandy’s older sister was given $2,400 and was told to keep the balance of the money after sharing with her siblings. Give Sandy exactly $350. Write Sandy’s portion

Answers

Sandy got 350 out of 2400.

Her portion is 350/2400 which can be reduced to:

35/240 = 7/48

The portion is 7/48

Solve for x and draw a number line. 3x−91>−87 AND 17x−16>18

Answers

Answer:

I hope this will help!

Step-by-step explanation:

PLEASE ANSWER QUICKLY ASAP
COMPLETE QUESTION B​

Answers

Answer:

Sector

Step-by-step explanation:

A sector of a circle is the portion of circle enclosed by two radii and arc

I will give brainliest to the right answer!! Find the vertex and the length of the latus rectum. x= 1/2 (y - 5)² + 7

Answers

Answer:

(7, 5)2

Step-by-step explanation:

When the quadratic is written in vertex form:

  x = a(y -k)^2 +h

the vertex is (x, y) = (h, k), and the length of the latus rectum is 1/a.

For your given equation, ...

  x = (1/2)(y -5)^2 +7

you have a=1/2, k = 5, h = 7, so ...

  the vertex is (7, 5)

  the length of the latus rectum is 1/(1/2) = 2

show that the point p(-6,2), Q(1,7) and R(6,3) are the vertices of scalene triangle​

Answers

Answer:

  the sides are different lengths as shown in the diagram

Step-by-step explanation:

Plotting the three points, you can see by "inspection" that the middle length side (PQ) is longer than the shortest side (QR) and shorter than the longest side (PR). You could use the distance formula to show this, or you can use a scale to measure the drawing.

A triangle with three unequal sides is a scalene triangle. ∆PQR is scalene.

Use the difference of squares identity to write this polynomial expression in factored form : 9x^2-49

Answers

Answer:

The expression in factored form is (3x - 7)(3x + 7)

Step-by-step explanation:

Here in this question, we are interested in using the difference of two squares to factor the given expression.

Mathematically, supposed we have two squares a^2 and b^2, and we are told to factorize a^2-b^2.

By using the difference of two squares;

a^2-b^2 can thus be written as;

(a-b)(a + b)

Now, we can apply same approach to the problem at hand.

9x^2 - 49

kindly note that 9x^2 can be written as ((3x)^2 and 49 can be written as 7^2

So applying what we have said earlier about difference of two squares;

9x^2 - 49 will be ;

(3x-7)(3x + 7)

Answer:

The answer is (3x - 7) (3x +7)

Step-by-step explanation:

n urn contains 3 red balls, 9 green, 2 yellow, 2 orange, and 4 purple balls. Two balls aredrawn, one at a time with replacement. Find the probability of drawing a green ball and an orangeball.

Answers

Answer:

[tex]\frac{9}{100}[/tex]

Step-by-step explanation:

Given:

Number of red balls, n(R) = 3

Number of green balls, n(G) = 9

Number of yellow balls, n(Y) = 2

Number of orange balls, n(O) = 2

Number of purple balls, n(P) = 4

Two balls are drawn one at a time with replacement.

To find:

Probability of drawing a green ball and an orange ball ?

Solution:

Total number of balls, n(Total) = 3 + 9 + 2 + 2 + 4 = 20

Formula for probability of an event E is given as:

[tex]P(E) = \dfrac{\text{Number of favorable cases}}{\text {Total number of cases}}[/tex]

Probability that a green ball is drawn at first:

[tex]P(Green) = \dfrac{\text{Number of Green balls}}{\text {Total number of Balls}}[/tex]

[tex]P(Green) = \dfrac{9}{20}[/tex]

Now, the ball is replaced , so total number of balls remain the same i.e. 20.[tex]P(Orange) = \dfrac{\text{Number of Orange balls}}{\text {Total number of Balls}}[/tex]

[tex]P(Orange) = \dfrac{2}{20} = \dfrac{1}{10}[/tex]

[tex]P(Green\ then\ orange) = P(Green) \times P(Orange)\\\Rightarrow P(Green\ then\ orange) = \dfrac{9}{10} \times \dfrac{1}{10}\\\Rightarrow P(Green\ then\ orange) = \bold{ \dfrac{9}{100} }[/tex]

In 2002, the population of a district was 22,800. With a continuous annual growth rate of approximately 5% what will the population be in 2012 according to the exponential growth function?

Answers

Answer:

37,139

Step-by-step explanation:

Given the following :

Population in 2002 = Initial population (P0) = 22,800

Growth rate (r) = 5% = 0.05

Growth in 2012 using the exponential growth function?

Time or period (t) = 2012 - 2002 = 10years

Exponential growth function:

P(t) = P0 * (1 + r) ^t

Where P(t) = population in t years

P(10) = 22800 * (1 + 0.05)^10

P(10) = 22800 * (1.05)^10

P(10) = 22800 * 1.62889

P(10) = 37138.797

P(10) = 37,139 ( to the nearest whole number)

Describe how to solve an absolute value equation
*will give brainliest*

Answers

Answer:

Step 1: Isolate the absolute value expression.

Step2: Set the quantity inside the absolute value notation equal to + and - the quantity on the other side of the equation.

Step 3: Solve for the unknown in both equations.

Step 4: Check your answer analytically or graphically.

Step-by-step explanation:

Answer:

Rewrite the absolute value equation as two separate equations, one positive and the other negative

Solve each equation separately

After solving, substitute your answers back into original equation to verify that you solutions are valid

Write out the final solution or graph it as needed

Step-by-step explanation:

What are the dimensions of the matrix?

Answers

The order of a matrix is m×n where m is the number of rows and n is the number of columns.

can you count and find what are m and n here?

Answer:

Step-by-step explanation:

Number of rows X Number of columns

Rows = 3

Columns = 2

answer = 3x2

For a ,a relationship to be a function, which values cannot repeat: the x-
values or the y-values? *

Answers

Answer:

              The  x - values

The y-values repeat in various functions (for example: quadratic function: y=x²; y=4 for x=2 and for x=-2)  

Please help quickly!!
A truck is driving up a hill with a 24% grade, so it climbs 24 feet vertically for every 100 feet horizontally.
What is the slope of the hill?

Answers

Answer:

6/25

Step-by-step explanation:

rise / run

24/100 = 6/25

Answer:

[tex]\frac{6}{25}[/tex]

Step-by-step explanation:

The slope of any relationship is always rise over run. This means the vertical distance traveled over the horizontal distance traveled will get us our slope.

We travels 24 feet vertically for every 100 feet horizontally, so:

[tex]\frac{24}{100}[/tex].

We can simplify this fraction to find the slope in fraction form.

[tex]\frac{24\div4}{100\div4} = \frac{6}{25}[/tex]

So the slope of this equation is [tex]\frac{6}{25}[/tex].

Hope this helped!

Calculate JK if LJ = 14, JM = 48, and LM = 50

Answers

Answer:

JK = 6.86

Step-by-step explanation:

The parameters given are;

LJ = 14

JM = 48

LM = 50

[tex]tan(\angle JML )= \dfrac{Opposite \ leg \ length}{Adjacent \ leg \ length} = \dfrac{LK}{JM} = \dfrac{14}{48} = \dfrac{7}{24}[/tex]

[tex]tan \left( \dfrac{7}{24} \right)= 16.26 ^{\circ }[/tex]

∠JML = 16.26°

Given that ∠JML is bisected by KM, we apply the angle bisector theorem which states that a ray that bisects an interior angle of a triangle bisects the opposite (bisected angle facing side) in the proportion of the ration of the other two sides of the triangle.

From the angle bisector theorem, we have;

LM/JM = LK/JK

50/48 = LK/JK................(1)

LK + KJ = 14.....................(2)

From equation (1), we have;

LK = 25/24×JK

25/24×KJ + JK = 14

JK×(25/24 + 1) = 14

JK × 49/24 = 14

JK = 14×24/49 = 48/7. = 6.86.

JK = 6.86

Other Questions
The discontinuity separating the crust from the mantle is named as: A.Mohorovicic B.Conrad C.Reppetty D.Gutemberg Land is purchased for $256,000. Additional costs include a $15,300 fee to a broker, a survey fee of $2,400, $1,750 to construct a fence, and a legal fee of $8,500. What is the cost of the land (x^2-4x)^2+7x^2-28x+12=0 According to the CAPM, what is the market risk premium given an expected return on a security of 15.8%, a stock beta of 1.1, and a risk-free interest rate of 7%? Multiple Choice 7.70% 6.05% 7.00% 8.00% Given: x - 5 > -2. Choose the solution set. Rubbing alcohol evaporates from your arm quickly, leaving a cooling effect on your skin. How do the molecules of gas compare to the molecules as a liquid? What did this person do to be considered a hero? In your answer, you should list at least three actions and briefly explain the effects. You might need to find outside sources to answer this question. Be sure to list the sources that you consulted during your research. A local mattress manufacturer wants to know if its manufacturing process is in or out of control and has hired you, a statistics expert in the field, to analyze its process. Specifically, the business has run 20 random samples of size 5 over the past month and has determined the mean of each sample. a. Determine the estimate of the mean when the process is in control. b. Assuming the process standard deviation is .50 and the mean of the process is the estimate calculated in part a, determine the Upper Control Limit (UCL) and the Lower Control Limit (LCL) for the manufacturing process. c. Explain the results to the vice-president of the mattress manufacturer focusing on whether, based on the results, the process is in or out of control. Sample no. Mean of Sample 1 95.72 2 95.44 3 95.40 4 95.50 5 95.56 6 95.72 7 95.60 8 95.24 9 95.46 10 95.44 11 95.80 12 95.20 13 94.82 14 95.78 15 95.18 16 95.32 17 95.08 18 95.22 19 95.04 20 95. EL PRESENTE PROGRESIVO: ( - ING) The spanish presente progresivo est formado con + . El participio presente is formed by adding to the stem of ar verbs and to the stem of er and -ir verbs. Oraciones Fill in the blanks with the present progressive form of the verbs and the correct verb conjugations. Do NOT use upper case letters in responses. 1. Gerardo, (entrenar) mucho hoy 2. Los estudiantes (sufrir) muchas presiones este semestre. 3. Yo (leer) mi libro favorito 4. t (traer) todos los libros de Matilde Casazola a clase? 5. Santiago y yo (abrir) una cafetera nueva hoy mismo. 6. Mis padres (ver) una pelcula nueva. 7. Yo (romper) mi vestido nuevo. 8. los estudiantes (dormir) pocas horas este semestre. According to this cartoon, what would giving women the right to vote do topolitics? How did WWI come to an end? On the first day in each month, Enid deposited $4 into her bank account and Jim deposited $3 into his. They opened these accounts on May 15, 1990. On December 31, 1990, they each had $72 dollars in their account. How much did each person deposit on May 15? Proteins in the cell membrane have many functions. Which type of protein would be used for cell recognition and as a receptor? A. Pore proteins B. Endoplasmic proteins C. Glycoproteins D. Integral proteins Which sentence in this excerpt from Ernest Hemingway's "In Another Country uses parallelism?In the fall the war was always there, but we did not go to it any more. It was cold in the fall in Milan and the dark came very early. Then theelectric lights came on, and it was pleasant along the streets looking in the windows. There was much game hanging outside the shops, and thesnow powdered in the fur of the foxes and the wind blew their tails. The deer hung stiff and heavy and empty, and small birds blew in the windand the wind turned their feathers. It was a cold fall and the wind came down from the mountains. hii guys ..help What does a text need for it to become a novel? Read the excerpt from A Poem for My Librarian, Mrs. Long.Mrs. Long asking what are you looking for todayWhen I wanted Leaves of Grass or alfred north whiteheadShe would go to the big library uptown and I now knowHat in hand to ask to borrow so that I might borrowThe implicit details in this excerpt best support the conclusion thatMrs. Long feels angry about having to ask to borrow books.the speaker asks for books that were often difficult to find.the speaker appreciates the librarians efforts to get her books.young children are not allowed to borrow books on their own. What is the mass number of an element 3(q7)=27 need help plzz 1st peep gets brainlest Suppose we want to test the color distribution claim on the M&Ms website that a bag of plain M&Ms is made up of 10% blue, 10% orange, 10% green, 20% red, 20% yellow, and 30% brown. We select a sample of 400 plain M&Ms and found the following: Color Blue Orange Green Red Yellow Brown Frequency 30 48 55 66 70 131 Is there evidence to doubt the color distribution claimed by the website? Use =0.05 URGENT PLZ HELP THANK YOU!